很激動,我終於看明白了 Dirichlet 定理的證明,完成了我多年來的夙願。證明過程我將以筆記的形式梳理出來,在此感謝 EM. Stein,以及他的名著《傅里葉分析導論》。

Dirichlet 定理

形如 qn+l 的等差數列中包含無數個素數,當 (q,l)=1 .

注意:字母 ql ,在接下來的論述中我們會繼續使用它在此處的含義。

證明思路

Dirichlet 引入了關鍵性的概念——特徵

  • 特徵分解

特徵是分析學中常見的概念,比如在實變函數中,利用特徵函數在可測集

上的識別作用,由此可以將簡單函數分解為典範形式,從而自然過渡到 Lebesgue 積分理論。

不過,Dirichlet 特徵獨特的地方在於,它是有限群 G=mathbb{Z}^*(q) 上的復值周期函數: G
ightarrow S_1 ,並且滿足完全可積性: e(ab)=e(a)e(b) ,這貨妥妥地與指數函數掛了鉤,於是傅里葉提刀霍霍而來。

我們先從一個最簡單的特徵函數開始:

[delta_{l}=egin{cases}  1&nequiv l   mod  q\  0&  otherwise end{cases}]

注意,我們不再簡簡單單地認為這是一個實值函數,而應該以復值函數的角度審視它。接下來主角上場——

[chi(m)=egin{cases}  e(m)&(m,q)=1\   0&  otherwise end{cases}]

特別地,我們稱 chi_0 稱為平凡特徵,

[chi_0(m)=egin{cases}  1&(m,q)=1\  0&  otherwise end{cases}]

Dirichlet 特徵,即 left{ e_0,e_1,...,e_{q-1} 
ight} 更是以復值函數空間的一組正交基而顯得尤為特別。這就意味著 delta_l 可以被諸 chi 們正交分解:

delta_l(m)=frac{1}{left| G 
ight|}sum_{chi}{hat delta_l(chi) chi(m)}

其中 hat delta_l(chi) delta_lchi 上的投影,並且已知 left| G 
ight|=varphi(q)

hat delta_l(chi)=frac{1}{varphi(q)}sum_{min G}{delta_l(chi) overline{chi(m)}}:=frac{overline{chi(l)}}{varphi(q)}

代入上式,

delta_l(m)=frac{1}{varphi(q)}sum_{chi}{overline{chi(l)}chi(m)} qquad (ast)

有了這個工具,就可以順理成章地將 Dirichlet 問題表達為解析形式。

  • 解析形式

一個通俗易懂但又不易為常人察覺的原理,若下面級數發散,則 Dirichlet 定理成立:

sum_{pequiv l , mod , q}{frac{1}{p}}

這是 Euler 大神的思路。這就需要我們通過特徵函數進一步分析問題的核心要點,

sum_{pequiv l , mod , q}{frac{1}{p^s}}=sum_{p}{frac{delta_l(p)}{p^s}}=frac{1}{varphi(q)}sum_{chi}{overline{chi(l)}sum_{p}{frac{chi(p)}{p^s}}}

我們需要考慮上式在復變數 s
ightarrow 1^+ 時的性態。此時,對上式的一個關鍵性的分解使得證明的目的變得明朗起來,

sum_{pequiv l , mod , q}{frac{1}{p^s}}=frac{1}{varphi(q)}left( sum_{p}{frac{chi_0(p)}{p^s}}+sum_{chi
echi_0}{overline{chi(l)}sum_{p}{frac{chi(p)}{p^s}}} 
ight)

其中括弧里第一項我們單拎出來發現:

sum_{p}{frac{chi_0(p)}{p^s}}=sum_{p ,not ,dividing ,q}{frac{1}{p^s}}
ightarrowinfty,s
ightarrow 1^+

關於這一點,實際上是利用了 zeta 函數的性質, s=1 是它的極點。已知能整除 q 的素數是有限的,所以剩下的不能整除 q 的倒數之和依然發散。

所以,如果能證明括弧里第二項有界,於是神功大成!因為 sum_{chi
echi_0} 求和只有有限多項,所以只需證明

left| sum_{p}{frac{chi(p)}{p^s}} 
ight|<infty

問題又來了,如何估計這一項呢?利用 chi 的完全可積性,可以得到和歐拉恆等式類似的形式:

sum_{n=1}^{infty}{frac{chi(n)}{n^s}} =prod_{p}frac{1}{1-chi(p)p^{-s}}

左式稱作 L-函數,記為 L(s,chi) ;兩邊同取對數(注意這裡取的對數乃是在複平面推廣的結果),

log L(s,chi)=-sum_{p}{log(1-chi(p)p^{-s})}

=sum_{p}{left[ frac{chi(p)}{p^s}+Oleft( frac{1}{p^{2s}} 
ight) 
ight]}

= sum_{p}{frac{chi(p)}{p^s}} +O(1)

於是問題轉化為 log L(s,chi) 是否有界,當 s
ightarrow 1^+ ,即 L(s,chi) 是否有界、非零——而事實上,這才是證明中最困難的地方。

證明

總而言之,一切的問題都歸結於L-函數的特性上,所以我們接下來重點研究它的性質。

  • L-函數

命題 1 q=p_1^{a_1}...p_N^{a_N} ,則

L(s,chi_0)=(1-p_1^{-s})...(1-p_N^{-s})zeta(s)

於是 lim_{s 
ightarrow 1^+}{L(s,chi_0)}=infty

通過前面的敘述已經十分明顯,證明略。

命題 2 L(s,chi) 有界,並且連續可導, 更進一步

(1) L(s,chi)=1+O(e^{-cs}) qquad s
ightarrow infty

(2) L(s,chi)=O(e^{-cs}) qquad s
ightarrow infty

想要證明該命題需要一個引理

引理 1 chi 非平凡,則

left| sum_{n=1}^{k}{chi(n)} 
ight|leq q ,qquad kin mathbb{Z}^+.

證明

對一個完整的周期求和,有

S:=sum_{n=1}^{q}{chi(n)}=0

這是因為取 ain mathbb{Z}^*(q) ,使得 chi (a)
e 1chi 非平凡保證了 a 的存在)

chi (a)S=sum_{n=1}^{q}{chi (a)chi(n)}=sum_{n=1}^{q}{chi(an)}=sum_{n=1}^{q}{chi(n)}=S

於是 S=0 ;若 k=aq+b0leq b <q

sum_{n=1}^{k}{chi(n)}=sum_{n=1}^{aq}{chi(n)}+sum_{aq < nleq aq+b}{chi(n)}=sum_{aq < nleq aq+b}{chi(n)}

即便將每一項的模放大到 1 ,亦不會超過 q

Q. E. D

繼續證明命題 2,利用 Abel 求和技巧

s_k:=sum_{n=1}^{k}{chi(n)}s_0=0

sum_{k=1}^{N}{frac{chi(k)}{k^s}} =sum_{k=1}^{N}{frac{s_k-s_{k-1}}{k^s}}

=sum_{k=1}^{N-1}s_kleft[ frac{1}{k^s}-frac{1}{(k+1)^s} 
ight]+frac{s_N}{N^s}

=sum_{k=1}^{N-1}f_k(s)+frac{s_N}{N^s}

其中 f_k(s)=s_kleft[ frac{1}{k^s}-frac{1}{(k+1)^s} 
ight] ,利用引理和中值定理得

left| f_k(s) 
ight|leq qsk^{-s-1}

由此, sum f_k(s) 絕對並且一致收斂,當 s>0 ;繼續估值

left| L(s,chi)-1 
ight|leq 2qsum_{n=2}^{infty}{n^{-s}}leq2^{-s}O(1)

同理也可證明第二個結論。

Q. E. D

只剩下最後一個命題就萬事大吉。

命題 3 chi
echi_0,則 L(1,chi)
e 0.

這個命題需要分為 chi 分別為復值與實值兩種情況。我們先證明復值的情況。

  • chi 取復值

這裡我們需要兩個引理來說明一個矛盾。

引理 2 s>1 ,則

prod_{chi}L(s,chi)geq1

此處對所有特徵求乘積.

證明

利用我們之前提到的恆等式

log L(s,chi)=-prod_{p}log(1-chi(p)p^{-s})

則原式可化為

log prod_{chi}L(s,chi)=-sum_{chi}sum_{p}log(1-chi(p)p^{-s})

=sum_{chi}sum_{p}sum_{k=1}^{infty}{frac{1}{k}}frac{chi(p^k)}{p^{ks}}

=sum_{p}sum_{k=1}^{infty}sum_{chi}{frac{1}{k}}frac{chi(p^k)}{p^{ks}}

又因為 sum_{chi}chi(p^k)=delta_0(p^k)varphi(q) (令 (ast) 式中的 l=0 即可得),於是

prod_{chi}L(s,chi)=expleft( varphi(q)sum_{p}sum_{k=1}^{infty}{frac{1}{k}}frac{delta_0(p^k)}{p^{ks}} 
ight)geq1

這是因為指數得每一項都是非負的。

Q. E. D

引理 3

(1) L(s,chi)=0,則 L(s,overlinechi)=0.

(2)chi
echi_0,且 L(1,chi)=0,則

left| L(s,chi) 
ight|leq Cleft| s-1 
ight| when  1leq s leq2

(3) 對於平凡特徵,

left| L(s,chi_0) 
ight|leq frac{C}{left| s-1 
ight|} when  1leq s leq2

證明

第一個結論顯然,這是因為 L(s,overlinechi)=overline{L(s,chi)}

第二個結論是因為由 L(s,chi) 的連續可導,當 s>0

最後一個結論成立的依據是命題 1 與關於 zeta 函數的一個估計:

zeta(s)leq 1+frac{1}{s-1} , s>1

Q. E. D

我們終於可以得到命題 3 的第一種情況的證明了。反證法(全程盯緊引理 3),假如存在某一個特徵 chi_1 ,有L(1,chi_1)=0,則其共軛亦為 0;考慮引理 2 中的連乘,連乘中唯一的無界因子是由平凡特徵所帶來的,其餘有限個特徵皆有界(命題 2),

prod_{chi}L(s,chi)=Ccdot L(s,chi_0)left| L(s,chi_1) 
ight|^2

leq frac{C}{left| s-1 
ight|}left| s-1 
ight|^2

=Cleft| s-1 
ight|
ightarrow 0s
ightarrow1^+

而這與引理 2 相矛盾。

  • chi 取實值

最後這種情況依舊使用反證法。在引出矛盾前,先要說明兩個符號:

F(m,n)=frac{chi(n)}{sqrt{mn}}

S_N=sum_{1leq mn leq N} F(m,n)

引理 4

(1) S_Ngeq c log Nc>0.

(2) S_N=2sqrt{N}L(1,chi)+O(1).

由此可見,若 L(1,chi)=0,則 (1)(2) 矛盾。

證明

我們沿雙曲線對 S_N 求和,注意到

sum_{mn=k}{frac{chi(n)}{sqrt{mn}}}=frac{1}{sqrt{k}}sum_{n |k}{chi(n)}

易證

[ sum_{n |k}{chi(n)}geqegin{cases}  1&k=d^2 \  0&  otherwise end{cases}]

代入於是得到

S_Ngeq sum_{1leq dleqsqrt{N}}{frac{1}{d}}geq clog N

最後的不等號由調和級數公式可得。下面證明第二個結論。

我們將 S_N 分為區域求和。

S_N=S_I+(S_{II}+S_{III})

S_I= sum_{mleqsqrt{N}}{frac{1}{sqrt{m}}}left( sum_{sqrt{N}<nleq N/m}{frac{chi (n)}{sqrt{n}}} 
ight)

S_{II}+S_{III}=sum_{nleqsqrt{N}}{frac{chi (n)}{sqrt{n}}}left( sum_{mleq N/n}{frac{1}{sqrt{m}}} 
ight)

由此我們發現三個級數估計亟待解決,事實上它們階——

 sum_{mleq N}{frac{1}{sqrt{m}}}=2sqrt{N}+c+Oleft( frac{1}{sqrt{N}} 
ight)

sum_{n=a}^{b}{frac{chi (n)}{sqrt{n}}}=Oleft( frac{1}{sqrt{a}} 
ight)

sum_{n=a}^{b}{frac{chi (n)}{n}}=Oleft( frac{1}{a} 
ight)

第一個估計容易證明,後兩個式子的估計借用依然利用 Abel 求和:

sum_{n=a}^{b}{frac{chi (n)}{sqrt{n}}}=sum_{n=a}^{b-1}{s_nleft[ frac{1}{sqrt{n}}-frac{1}{sqrt{n+1}} 
ight]}+Oleft( frac{1}{sqrt{a}} 
ight)

=Oleft( sum_{n=a}^{infty}{frac{1}{n^{3/2}}} 
ight)+Oleft( frac{1}{sqrt{a}} 
ight)

=Oleft( frac{1}{sqrt{a}} 
ight)

將估計結果代入 S_IS_{II}+S_{III} 中,得到

S_I= O(N^{1/4}N^{-1/4})=O(1)

S_{II}+S_{III}=2sqrt{N}sum_{nleq sqrt{N}}^{b}{frac{chi (n)}{n}}+csum_{nleq sqrt{N}}{frac{chi (n)}{sqrt{n}}}+Oleft( frac{1}{sqrt{N}}sum_{nleq sqrt{N}}{1} 
ight)

=A+B+C

A=2sqrt{N}L(1,chi)+O(N^{1/2}N^{-1/2})

B=O(1)

C=O(1)

於是引理 4 成立,也就是說我們得到了命題 3 的所有情況,Dirichlet 定理證畢!


推薦閱讀:
相关文章